Strona 19 z 94

[Rozgrzewka OM][MIX][Nierówności] Nierówności

: 4 gru 2009, o 23:20
autor: Swistak
No to tu macie:

1 raczej trywialna, 3 raczej cięższa (tzn. ja bym na pewno nie zrobił, ale mamy tutaj niezłych rozkminiaczy i dla nich chyba taka trudna nie będzie ).

[Rozgrzewka OM][MIX][Nierówności] Nierówności

: 4 gru 2009, o 23:50
autor: Piotr Rutkowski
Dumel pisze:ok może coś takiego:
\(\displaystyle{ \frac{1}{ \sqrt{4n} } \le \frac{1}{2}\cdot \frac{3}{4} \cdot \ldots \cdot \frac{2n-1}{2n}< \frac{1}{ \sqrt{2n} }}\)
Wystarczy dorobić pomocniczy ciąg, który bedzie nam ładnie zwijał iloczyn...
Co do tej "trudniejszej" nierówności nr 3, to sprowadza się ona do pomnożenia przez mianownik po prawej stronie, pomnożenia przez 2 i zastosowania AM-GM...

[Rozgrzewka OM][MIX][Nierówności] Nierówności

: 5 gru 2009, o 11:37
autor: Dumel
Wystarczy dorobić pomocniczy ciąg, który bedzie nam ładnie zwijał iloczyn...
wrzuciłem ją z myślą o początkujących, aby zobaczyli
Ukryta treść:    

[Rozgrzewka OM][MIX][Nierówności] Nierówności

: 6 gru 2009, o 00:01
autor: pawelsuz
Piotr Rutkowski, mógłbyś pokazać to rozwiązanie nierówności z 3 zad z tego linka?
I tak w ogóle to była umowa, że wrzucamy całe rozwiązania, a nie rzucamy pomysłami, więc można by się tego trzymać...

[Rozgrzewka OM][MIX][Nierówności] Nierówności

: 6 gru 2009, o 00:27
autor: limes123
Mamy po przemnozeniu stronami \(\displaystyle{ \sum\frac{abc+1}{ab+a}\geq 3}\) i dodajemy sobie stronami po trzy jedynki. Dostajemy \(\displaystyle{ \sum\frac{abc+ab+a+1}{ab+a}\geq 6\iff \sum \frac{b(c+1)}{a+1}+\sum\frac{a+1}{a(b+1)}\geq 3(\sqrt[3]{abc}+\frac{1}{\sqrt[3]{abc}})\geq 6}\).

[Rozgrzewka OM][MIX][Nierówności] Nierówności

: 6 gru 2009, o 00:47
autor: pawelsuz
Dzięki:d

[Rozgrzewka OM][MIX][Nierówności] Nierówności

: 6 gru 2009, o 11:18
autor: Piotr Rutkowski
Albo (zupełnie analogicznie):
\(\displaystyle{ 2\sum_{cyc}\frac{1+abc}{a(b+1)}=\sum_{cyc}\frac{1+a+abc+ab}{a(b+1)}=\sum_{cyc}(\frac{1+a}{a(b+1)}+\frac{ab(c+1)}{b+1})=\sum_{cyc}(\frac{1+a}{a(b+1)}+\frac{b(c+1)}{b+1})\geq 6\sqrt[6]{\prod_{cyc}(\frac{1+a}{a(b+1)}\cdot \frac{b(c+1)}{b+1})}=6}\)

W takim razie trzymając się konwencji czuję się zobligowany do podania rozwiązania także do tamtego zadanka:

Niech \(\displaystyle{ a_{n}=\frac{2n-1}{2n}}\). Teraz zdefiniujmy \(\displaystyle{ b_{n}}\) w następujący sposób:
-\(\displaystyle{ b_{1}=1}\)
-\(\displaystyle{ \forall_{n\geq 2} \ b_{n}=\frac{2n-2}{2n-1}}\)
Zauważmy, że zachodzą następujące nierówności:
1) \(\displaystyle{ (a_{n}\leq b_{n})\iff (a_{n}\leq \sqrt{a_{n}b_{n}})}\)
2) \(\displaystyle{ (a_{n}\geq b_{n+1})\iff (a_{n}\geq \sqrt{a_{n}b_{n+1}})}\)
Teraz mnożąc te nierówności odpowiednią ilość razy otrzymamy:
\(\displaystyle{ \frac{1}{\sqrt{4n}}<\frac{1}{\sqrt{2n+1}}=\sqrt{\prod_{i=1}^{n}a_{n}b_{n+1}}=\prod_{i=1}^{n}\sqrt{a_{n}b_{n+1}}\leq \prod_{i=1}^{n}a_{n}<\prod_{i=1}^{n}\sqrt{a_{n}b_{n}}=\sqrt{\prod_{i=1}^{n}a_{n}b_{n}}=\frac{1}{\sqrt{2n}}}\)
Pozdrawiam

[Rozgrzewka OM][MIX][Nierówności] Nierówności

: 6 gru 2009, o 14:17
autor: pawelsuz
Niech \(\displaystyle{ a,b,c}\) będą bokami trójkąta, a \(\displaystyle{ s}\) jego polem. Udowodnić, że zachodzi:
\(\displaystyle{ a^2+b^2+c^2 \ge 4 \sqrt{3} s}\)

[Rozgrzewka OM][MIX][Nierówności] Nierówności

: 6 gru 2009, o 14:42
autor: Piotr Rutkowski
Standardowe podstawienie \(\displaystyle{ a=x+y ...}\) itd. w połączeniu z wzorem Herona daje nam:
\(\displaystyle{ \sum_{cyc}(x+y)^{2}\geq 4\sqrt{3}\sqrt{(x+y+z)xyz}}\) co można udowodnić na milion sposobów np.:
\(\displaystyle{ \sum_{cyc}(x+y)^{2}\geq 4\sum_{cyc}xy=4\sqrt{(\sum_{cyc}xy)^{2}}= 4\sqrt{(\sum_{cyc}(xy)^{2})+2(\sum_{cyc}xy\cdot yz)}\geq 4\sqrt{3\sum_{cyc}xy\cdot yx}=4\sqrt{3}\sqrt{(x+y+z)xyz}}\)
Q.E.D.

[Rozgrzewka OM][MIX][Nierówności] Nierówności

: 6 gru 2009, o 14:50
autor: jerzozwierz
Można inaczej.
\(\displaystyle{ \frac{a^{2}+b^{2}+c^{2}}{2S} \ge \frac{ab+bc+ca}{2S}= \frac{1}{\sin \alpha}+ \frac{1}{\sin \beta}+ \frac{1}{\sin \gamma} \ge 2 \sqrt{3}}\) z Jensena.

[Rozgrzewka OM][MIX][Nierówności] Nierówności

: 6 gru 2009, o 15:46
autor: binaj
Liczy rzeczywiste \(\displaystyle{ x_1,x_2...x_n}\) spełniają warunek: \(\displaystyle{ x_1+x_2+...+x_n=0}\)

Niech \(\displaystyle{ m}\) będzie najmniejszą z tych liczb, a \(\displaystyle{ M}\) największą.

Dowieść, że zachodzi: \(\displaystyle{ x_1^2+x_2^2+...+x_n^2 \le -nmM}\)

[Rozgrzewka OM][MIX][Nierówności] Nierówności

: 6 gru 2009, o 17:57
autor: Dumel
raczej proste, ale przy pisaniu rozwiązania ciągle m myliło mi się z n - to taki ukryty haczyk więc uważajcie

[Rozgrzewka OM][MIX][Nierówności] Nierówności

: 28 gru 2009, o 20:13
autor: inny
Dumel może wrzuć rozwiązanie, bo temat stoi..

[Rozgrzewka OM][MIX][Nierówności] Nierówności

: 28 gru 2009, o 23:38
autor: binaj
wskazówka:    

[Rozgrzewka OM][MIX][Nierówności] Nierówności

: 29 gru 2009, o 00:37
autor: pawelsuz
No teraz to już banał:P
Zgodnie z wskazówką, tworzymy n takich nierówności, wymnażamy przez nawiasy i dodajemy stronami otrzymując
\(\displaystyle{ M( \sum_{i=1}^{n}x_{i})+m(\sum_{i=1}^{n}x_{i})-nmM \ge \sum_{i=1}^{n}x_{i}^{2}}\)
Korzystając z założenia \(\displaystyle{ \sum_{i=1}^{n}x_{i}=0}\) mamy tezę.

-- 29 grudnia 2009, 00:47 --

Dane są liczby dodatnie spełniające nierówność
\(\displaystyle{ \sqrt{\frac{bcd}{a}}+\sqrt{\frac{acd}{b}}+\sqrt{\frac{abd}{c}}+\sqrt{\frac{abc}{d}}\leq (a+b)(c+d)}\)
Udowodnić, że prawdziwa jest nierówność:
\(\displaystyle{ \frac{bd}{a}+\frac{ac}{b}+\frac{ad}{c}+\frac{bc}{d}\geq\frac{1}{a}+\frac{1}{b}+\frac{1}{c}+\frac{1}{d}}\)